Secuencia A000607 en la Enciclopedia Online de Secuencias de Números Enteros es el número de particiones de $n$ en partes principales. Por ejemplo, hay $5$ particiones de $10$ en partes principales: $10 = 2 + 2 + 2 + 2 + 2 = 2 + 2 + 3 + 3 = 2 + 3 + 5 = 3 + 7 = 5 + 5.$ La OEIS da una expresión asintótica
$$A000607(n) \sim \exp\left(2 \pi \sqrt{\frac{n}{3 \log n}}\right). $$
Numéricamente, esto parece ser incorrecto incluso si se toma el logaritmo de ambos lados. Mi conjetura es que
$$\lim_{n \to \infty} \log\left(A000607(n)\right) \bigg/ \left( 2 \pi \sqrt{\frac{n}{3 \log n}} \right) \ne 1.$$
Véase el siguiente gráfico:
¿Cómo se puede probar o refutar esta conjetura?
Para más referencias, consulte http://oeis.org/A000607 .
0 votos
Hay un debate sobre las particiones en partes primarias en math.stackexchange.com/questions/89240/prime-partition (pero no se discute específicamente la asintótica).
3 votos
Más relevante es la discusión en mathoverflow.net/questions/69680/ donde se cita el libro de Flajolet y Sedgewick que tiene una prueba de (la forma logarítmica de) la asintótica citada de oeis.